www.vorhilfe.de
Vorhilfe

Kostenlose Kommunikationsplattform für gegenseitige Hilfestellungen.
Hallo Gast!einloggen | registrieren ]
Startseite · Forum · Wissen · Kurse · Mitglieder · Team · Impressum
Forenbaum
^ Forenbaum
Status Englisch
  Status Grammatik
  Status Lektüre
  Status Korrekturlesen
  Status Übersetzung
  Status Sonstiges (Englisch)

Gezeigt werden alle Foren bis zur Tiefe 2

Navigation
 Startseite...
 Neuerdings beta neu
 Forum...
 vorwissen...
 vorkurse...
 Werkzeuge...
 Nachhilfevermittlung beta...
 Online-Spiele beta
 Suchen
 Verein...
 Impressum
Das Projekt
Server und Internetanbindung werden durch Spenden finanziert.
Organisiert wird das Projekt von unserem Koordinatorenteam.
Hunderte Mitglieder helfen ehrenamtlich in unseren moderierten Foren.
Anbieter der Seite ist der gemeinnützige Verein "Vorhilfe.de e.V.".
Partnerseiten
Weitere Fächer:

Open Source FunktionenplotterFunkyPlot: Kostenloser und quelloffener Funktionenplotter für Linux und andere Betriebssysteme
Forum "Stetigkeit" - Minimum/Maximum bestimmen
Minimum/Maximum bestimmen < Stetigkeit < Funktionen < eindimensional < reell < Analysis < Hochschule < Mathe < Vorhilfe
Ansicht: [ geschachtelt ] | ^ Forum "Stetigkeit"  | ^^ Alle Foren  | ^ Forenbaum  | Materialien

Minimum/Maximum bestimmen: Frage (beantwortet)
Status: (Frage) beantwortet Status 
Datum: 14:50 Do 17.01.2013
Autor: Blackburn4717537

Aufgabe
Man betrachte die Funktion

f: [0, [mm] +\infty) \to \IR, [/mm] f(x) = [mm] \bruch{\wurzel[3]{x^2+1}+\wurzel[3]{x}}{\wurzel{x^3+2}} [/mm]

Besitzt f ein Minimum? Besitzt f ein Maximum?


Hallo zusammen,

meine Idee hierzu ist die folgende:

Wir zeigen zuerst, dass f stetig ist.
Dann betrachten wir ein abgeschlossenes und beschränktes Intervall [0;a] [mm] \subset [0;+\infty), [/mm] wobei a eine positive reelle Zahl ist.
Mit dem Satz von Minimum und Maximum nach Weierstraß können wir dann folgern, dass f eingeschränkt auf [0;a] ein Minimum und Maximum annimmt für jedes positive a.
Hieraus möchte ich dann irgendwie folgern, dass f ein Minimum/Maximum annimmt auf [mm] [0,+\infty) [/mm] oder auch nicht.

Was meint ihr dazu?

Gruss
Alexander

EDIT:

Ich habe mir jetzt den Graphen mal zeichnen lassen, und f besitzt ein Maximum, aber kein Minimum. Wie zeige ich jetzt am besten, dass der Grenzwert für x [mm] \to +\infty [/mm] 0 ist?

        
Bezug
Minimum/Maximum bestimmen: Mitteilung
Status: (Mitteilung) Reaktion unnötig Status 
Datum: 18:56 Do 17.01.2013
Autor: Blackburn4717537

Niemand?

Bezug
        
Bezug
Minimum/Maximum bestimmen: Antwort
Status: (Antwort) fertig Status 
Datum: 19:44 Do 17.01.2013
Autor: Marcel

Hallo,

> Man betrachte die Funktion
>  
> f: [0, [mm]+\infty) \to \IR,[/mm] f(x) =
> [mm]\bruch{\wurzel[3]{x^2+1}+\wurzel[3]{x}}{\wurzel{x^3+2}}[/mm]
>  
> Besitzt f ein Minimum? Besitzt f ein Maximum?
>  
> Hallo zusammen,
>  
> meine Idee hierzu ist die folgende:
>  
> Wir zeigen zuerst, dass f stetig ist.
>  Dann betrachten wir ein abgeschlossenes und beschränktes
> Intervall [0;a] [mm]\subset [0;+\infty),[/mm] wobei a eine positive
> reelle Zahl ist.
>  Mit dem Satz von Minimum und Maximum nach Weierstraß
> können wir dann folgern, dass f eingeschränkt auf [0;a]
> ein Minimum und Maximum annimmt für jedes positive a.
>  Hieraus möchte ich dann irgendwie folgern, dass f ein
> Minimum/Maximum annimmt auf [mm][0,+\infty)[/mm] oder auch nicht.
>  
> Was meint ihr dazu?
>  
> Gruss
>  Alexander
>  
> EDIT:
>  
> Ich habe mir jetzt den Graphen mal zeichnen lassen,

das darf/kann man in der Klausur nicht, aber genau das wollte ich Dir
gerade vorschlagen.

> und f
> besitzt ein Maximum, aber kein Minimum. Wie zeige ich jetzt
> am besten, dass der Grenzwert für x [mm]\to +\infty[/mm] 0 ist?

Wenn Du das brauchst: Ob das nun auch mit de l'Hôpital geht, kann man
sich ja auch mal ergänzend überlegen. Es geht jedenfalls auch relativ
einfach:
[mm] $$\bruch{\wurzel[3]{x^2+1}+\wurzel[3]{x}}{\wurzel{x^3+2}}=\bruch{\wurzel[6]{(x^2+1)^2}+\wurzel[6]{x^2}}{\wurzel{x^3+2}}=\frac{\wurzel[6]{\tfrac{x^9}{x^9}(x^2+1)^2}+\wurzel[6]{\tfrac{x^9}{x^9}x^2}}{x^{3/2}*\sqrt{1+\tfrac{2}{x^3}}}=\frac{\wurzel[6]{x^9*\tfrac{(x^2+1)^2}{x^9}}+\wurzel[6]{x^9*\tfrac{x^2}{x^9}}}{x^{3/2}*\sqrt{1+\tfrac{2}{x^3}}}=...$$ [/mm]

Vielleicht siehst Du auch alleine, wie's weiter geht?

Übrigens, mal nebenbei: Du kannst hier sicher einfach mit [mm] $f\,'$ [/mm] ein lokales
Maximum bestimmen, oder? Und anhand von [mm] $f\,'$ [/mm] begründest Du dann,
dass "links" von der lokalen Maximalstelle [mm] $f\,$ [/mm] monoton wachsend ist,
und das "rechts" von der lokalen Maximalstelle [mm] $f\,$ [/mm] monoton fallend ist.
Damit kannst Du dann leicht folgern, dass diese lokale Maximalstelle auch
eine globale ist. Und das [mm] $\inf [/mm] f=0$ ist, folgt aus obiger
Grenzwertbetrachtung (es ist ja $f [mm] \ge [/mm] 0$ klar). Danach geht's dann weiter:
Angenommen, es gäbe eine Stelle [mm] $x_m \ge 0\,,$ [/mm] an der [mm] $f\,$ [/mm] sein Infimum
annimmt, also [mm] $f(x_m)=0\,.$ [/mm] Dann folgte...

und führst das zum Widerspruch.

Gruß,
  Marcel

Bezug
                
Bezug
Minimum/Maximum bestimmen: Frage (beantwortet)
Status: (Frage) beantwortet Status 
Datum: 20:55 Do 17.01.2013
Autor: Blackburn4717537


> das darf/kann man in der Klausur nicht, aber genau das
> wollte ich Dir
> gerade vorschlagen.

Ja, das stimmt, aber nach einiger Zeit hatte ich dann die Schn.... voll, und habe mir den Graphen dann einfach zeichnen lassen. Ich hoffe mal nicht, dass in unserer Klausur so eine Aufgabe dran kommt...

> Vielleicht siehst Du auch alleine, wie's weiter geht?

de l'Hôpital kenne ich zwar aus der Schule, aber hatten wir in der Vorlesung noch nicht.

Ich habe mal weiter umgeformt, und folgendes rausbekommen:

[mm] \bruch{\wurzel[3]{x^2+1}+\wurzel[3]{x}}{\wurzel{x^3+2}}=\bruch{\wurzel[6]{(x^2+1)^2}+\wurzel[6]{x^2}}{\wurzel{x^3+2}}=\frac{\wurzel[6]{\tfrac{x^9}{x^9}(x^2+1)^2}+\wurzel[6]{\tfrac{x^9}{x^9}x^2}}{x^{3/2}*\sqrt{1+\tfrac{2}{x^3}}}=\frac{\wurzel[6]{x^9*\tfrac{(x^2+1)^2}{x^9}}+\wurzel[6]{x^9*\tfrac{x^2}{x^9}}}{x^{3/2}*\sqrt{1+\tfrac{2}{x^3}}}=\bruch{\wurzel[6]{x^9}\wurzel[6]{\bruch{(x^2+1)^2}{x^9}}+\wurzel[6]{x^9}\wurzel[6]{\bruch{x^2}{x^9}}}{x^{3/2}\wurzel{1+\bruch{2}{x^3}}}=\bruch{\wurzel[6]{\bruch{(x^2+1)^2}{x^9}}+\wurzel[6]{\bruch{x^2}{x^9}}}{\wurzel{1+\bruch{2}{x^3}}} \to [/mm] 0 für x [mm] \to +\infty [/mm]

> Übrigens, mal nebenbei: Du kannst hier sicher einfach mit
> [mm]f\,'[/mm] ein lokales
>  Maximum bestimmen, oder? Und anhand von [mm]f\,'[/mm] begründest
> Du dann,
>  dass "links" von der lokalen Maximalstelle [mm]f\,[/mm] monoton
> wachsend ist,
> und das "rechts" von der lokalen Maximalstelle [mm]f\,[/mm] monoton
> fallend ist.
>  Damit kannst Du dann leicht folgern, dass diese lokale
> Maximalstelle auch
> eine globale ist. Und das [mm]\inf f=0[/mm] ist, folgt aus obiger
> Grenzwertbetrachtung (es ist ja [mm]f \ge 0[/mm] klar). Danach
> geht's dann weiter:
> Angenommen, es gäbe eine Stelle [mm]x_m \ge 0\,,[/mm] an der [mm]f\,[/mm]
> sein Infimum
> annimmt, also [mm]f(x_m)=0\,.[/mm] Dann folgte...
>  
> und führst das zum Widerspruch.

Ableitungen hatten wir leider noch nicht.

Meine Idee ist die folgende:

Es ist f(0) = [mm] \bruch{\wurzel{2}}{2}\approx [/mm] 0,7071 und
[mm] f(1)\approx [/mm] 1,3048

Also gibt es auf jeden Fall ein [mm] x_0 \in (0,+\infty), [/mm] sodass [mm] f(x_0) [/mm] > f(0).
Nun wissen wir aber auch, dass f(x) [mm] \to [/mm] 0 für x [mm] \to +\infty. [/mm]
Mithin folgt, dass f ein Maximum besitzen muss.

Behauptung: f besitzt kein Minimum, da:

Offensichtlich ist f positiv für alle x. Wegen f(x) [mm] \to [/mm] 0 für x [mm] \to +\infty, [/mm] ist das inf f = 0. Es gibt aber kein [mm] x_1 \in [0,+\infty), [/mm] sodass [mm] f(x_1) [/mm] = 0, da f immer positiv für alle x ist. Also kann inf f nicht im Bild von f liegen, und somit hat f kein Minimum.

Reicht das als Beweis?

Gruss
Alexander

Bezug
                        
Bezug
Minimum/Maximum bestimmen: Antwort
Status: (Antwort) fertig Status 
Datum: 00:41 Fr 18.01.2013
Autor: Helbig

Hallo Alexander,

> > das darf/kann man in der Klausur nicht, aber genau das
> > wollte ich Dir
> > gerade vorschlagen.
>  
> Ja, das stimmt, aber nach einiger Zeit hatte ich dann die
> Schn.... voll, und habe mir den Graphen dann einfach
> zeichnen lassen. Ich hoffe mal nicht, dass in unserer
> Klausur so eine Aufgabe dran kommt...
>  
> > Vielleicht siehst Du auch alleine, wie's weiter geht?
>  
> de l'Hôpital kenne ich zwar aus der Schule, aber hatten
> wir in der Vorlesung noch nicht.
>  
> Ich habe mal weiter umgeformt, und folgendes rausbekommen:
>  
> [mm]\bruch{\wurzel[3]{x^2+1}+\wurzel[3]{x}}{\wurzel{x^3+2}}=\bruch{\wurzel[6]{(x^2+1)^2}+\wurzel[6]{x^2}}{\wurzel{x^3+2}}=\frac{\wurzel[6]{\tfrac{x^9}{x^9}(x^2+1)^2}+\wurzel[6]{\tfrac{x^9}{x^9}x^2}}{x^{3/2}*\sqrt{1+\tfrac{2}{x^3}}}=\frac{\wurzel[6]{x^9*\tfrac{(x^2+1)^2}{x^9}}+\wurzel[6]{x^9*\tfrac{x^2}{x^9}}}{x^{3/2}*\sqrt{1+\tfrac{2}{x^3}}}=\bruch{\wurzel[6]{x^9}\wurzel[6]{\bruch{(x^2+1)^2}{x^9}}+\wurzel[6]{x^9}\wurzel[6]{\bruch{x^2}{x^9}}}{x^{3/2}\wurzel{1+\bruch{2}{x^3}}}=\bruch{\wurzel[6]{\bruch{(x^2+1)^2}{x^9}}+\wurzel[6]{\bruch{x^2}{x^9}}}{\wurzel{1+\bruch{2}{x^3}}} \to[/mm]
> 0 für x [mm]\to +\infty[/mm]
>  
> > Übrigens, mal nebenbei: Du kannst hier sicher einfach mit
> > [mm]f\,'[/mm] ein lokales
>  >  Maximum bestimmen, oder? Und anhand von [mm]f\,'[/mm]
> begründest
> > Du dann,
>  >  dass "links" von der lokalen Maximalstelle [mm]f\,[/mm] monoton
> > wachsend ist,
> > und das "rechts" von der lokalen Maximalstelle [mm]f\,[/mm] monoton
> > fallend ist.
>  >  Damit kannst Du dann leicht folgern, dass diese lokale
> > Maximalstelle auch
> > eine globale ist. Und das [mm]\inf f=0[/mm] ist, folgt aus obiger
> > Grenzwertbetrachtung (es ist ja [mm]f \ge 0[/mm] klar). Danach
> > geht's dann weiter:
> > Angenommen, es gäbe eine Stelle [mm]x_m \ge 0\,,[/mm] an der [mm]f\,[/mm]
> > sein Infimum
> > annimmt, also [mm]f(x_m)=0\,.[/mm] Dann folgte...
>  >  
> > und führst das zum Widerspruch.
>  
> Ableitungen hatten wir leider noch nicht.
>  
> Meine Idee ist die folgende:
>  
> Es ist f(0) = [mm]\bruch{\wurzel{2}}{2}\approx[/mm] 0,7071 und
>  [mm]f(1)\approx[/mm] 1,3048
>  
> Also gibt es auf jeden Fall ein [mm]x_0 \in (0,+\infty),[/mm] sodass
> [mm]f(x_0)[/mm] > f(0).
>  Nun wissen wir aber auch, dass f(x) [mm]\to[/mm] 0 für x [mm]\to +\infty.[/mm]
>  
> Mithin folgt, dass f ein Maximum besitzen muss.

Das ist jetzt mehr anschaulich.
Ich würde so argumentieren:

Wegen $f(0) > 0$ und [mm] $f(x)\to [/mm] 0$ für [mm] $x\to\infty$ [/mm] gibt es ein [mm] $a\in(0;\infty)$ [/mm] mit $f(x) < f(0)$ für alle [mm] $x>a\,.$ [/mm] Weil $f$ auf dem Kompaktum $[0; a]$ stetig ist, nimmt es sein Maximum auf diesem Kompaktum in einem Punkt [mm] $x_0$ [/mm] an. Nun ist [mm] $f(x_0) \ge [/mm] f(0) > f(x)$ für alle [mm] $x\in (a;\infty)$ [/mm] und nach Konstruktion ist [mm] $f(x_0)\ge [/mm] f(x)$ für alle [mm] $x\in [0;a]\;.$ [/mm] Damit ist [mm] $f(x_0)$ [/mm] das Maximum von $f$ auf [mm] $[0,\infty)$. [/mm]

Übrigens kann man [mm] $f(x)\to [/mm] 0$ etwas einfacher zeigen: Für $x>1$ ist

   $0< [mm] {\root 3\of {x^2+1} + \root 3 \of x \over \sqrt {x^3+2}}<{\sqrt {x^2+1} + \sqrt x \over \sqrt {x^3+2}} \to [/mm] 0$ für [mm] $x\to\infty\,.$ [/mm]


>  
> Behauptung: f besitzt kein Minimum, da:
>  
> Offensichtlich ist f positiv für alle x. Wegen f(x) [mm]\to[/mm] 0
> für x [mm]\to +\infty,[/mm] ist das inf f = 0. Es gibt aber kein
> [mm]x_1 \in [0,+\infty),[/mm] sodass [mm]f(x_1)[/mm] = 0, da f immer positiv
> für alle x ist. Also kann inf f nicht im Bild von f
> liegen, und somit hat f kein Minimum.

Richtig!

Gruß,
Wolfgang



Bezug
                                
Bezug
Minimum/Maximum bestimmen: Frage (beantwortet)
Status: (Frage) beantwortet Status 
Datum: 12:45 Sa 19.01.2013
Autor: Blackburn4717537

Hallo Wolfgang,

> Das ist jetzt mehr anschaulich.
>  Ich würde so argumentieren:
>  
> Wegen [mm]f(0) > 0[/mm] und [mm]f(x)\to 0[/mm] für [mm]x\to\infty[/mm] gibt es ein
> [mm]a\in(0;\infty)[/mm] mit [mm]f(x) < f(0)[/mm] für alle [mm]x>a\,.[/mm] Weil [mm]f[/mm] auf
> dem Kompaktum [mm][0; a][/mm] stetig ist, nimmt es sein Maximum auf
> diesem Kompaktum in einem Punkt [mm]x_0[/mm] an. Nun ist [mm]f(x_0) \ge f(0) > f(x)[/mm]
> für alle [mm]x\in (a;\infty)[/mm] und nach Konstruktion ist
> [mm]f(x_0)\ge f(x)[/mm] für alle [mm]x\in [0;a]\;.[/mm] Damit ist [mm]f(x_0)[/mm] das
> Maximum von [mm]f[/mm] auf [mm][0,\infty)[/mm].

Okay.
  

> Übrigens kann man [mm]f(x)\to 0[/mm] etwas einfacher zeigen: Für
> [mm]x>1[/mm] ist
>  
> [mm]0< {\root 3\of {x^2+1} + \root 3 \of x \over \sqrt {x^3+2}}<{\sqrt {x^2+1} + \sqrt x \over \sqrt {x^3+2}} \to 0[/mm]
> für [mm]x\to\infty\,.[/mm]

Okay, die Abschätzung leuchtet ein.
Ich denke mal die Begründung ist dann die folgende:
Im Zähler und Nenner stehen dieselben Wurzeln. Im Zähler haben wir [mm] \sqrt{x^2+1} [/mm] + [mm] \sqrt{x} [/mm] stehen. Für x [mm] \to +\infty [/mm] geht die Summe gegen [mm] +\infty. [/mm]
Im Nenner haben wir jedoch [mm] \sqrt {x^3+2} [/mm] stehen, was aber auch gegen [mm] +\infty [/mm] geht. Nun geht jedoch [mm] x^3 [/mm] schneller gegen [mm] +\infty, [/mm] als das was im Zähler steht, also geht der Bruch gegen 0.
Würde das in einer Klausur auch so als Begründung reichen?

Gruss
Alexander

Bezug
                                        
Bezug
Minimum/Maximum bestimmen: Antwort
Status: (Antwort) fertig Status 
Datum: 13:03 Sa 19.01.2013
Autor: Helbig

Hallo Alexander,

> Hallo Wolfgang,
>  
> > Das ist jetzt mehr anschaulich.
>  >  Ich würde so argumentieren:
>  >  
> > Wegen [mm]f(0) > 0[/mm] und [mm]f(x)\to 0[/mm] für [mm]x\to\infty[/mm] gibt es ein
> > [mm]a\in(0;\infty)[/mm] mit [mm]f(x) < f(0)[/mm] für alle [mm]x>a\,.[/mm] Weil [mm]f[/mm] auf
> > dem Kompaktum [mm][0; a][/mm] stetig ist, nimmt es sein Maximum auf
> > diesem Kompaktum in einem Punkt [mm]x_0[/mm] an. Nun ist [mm]f(x_0) \ge f(0) > f(x)[/mm]
> > für alle [mm]x\in (a;\infty)[/mm] und nach Konstruktion ist
> > [mm]f(x_0)\ge f(x)[/mm] für alle [mm]x\in [0;a]\;.[/mm] Damit ist [mm]f(x_0)[/mm] das
> > Maximum von [mm]f[/mm] auf [mm][0,\infty)[/mm].
>  
> Okay.
>    
> > Übrigens kann man [mm]f(x)\to 0[/mm] etwas einfacher zeigen: Für
> > [mm]x>1[/mm] ist
>  >  
> > [mm]0< {\root 3\of {x^2+1} + \root 3 \of x \over \sqrt {x^3+2}}<{\sqrt {x^2+1} + \sqrt x \over \sqrt {x^3+2}} \to 0[/mm]
> > für [mm]x\to\infty\,.[/mm]
>  
> Okay, die Abschätzung leuchtet ein.
>  Ich denke mal die Begründung ist dann die folgende:
>  Im Zähler und Nenner stehen dieselben Wurzeln. Im Zähler
> haben wir [mm]\sqrt{x^2+1}[/mm] + [mm]\sqrt{x}[/mm] stehen. Für x [mm]\to +\infty[/mm]
> geht die Summe gegen [mm]+\infty.[/mm]
>  Im Nenner haben wir jedoch [mm]\sqrt {x^3+2}[/mm] stehen, was aber
> auch gegen [mm]+\infty[/mm] geht. Nun geht jedoch [mm]x^3[/mm] schneller
> gegen [mm]+\infty,[/mm] als das was im Zähler steht, also geht der
> Bruch gegen 0.
>  Würde das in einer Klausur auch so als Begründung
> reichen?

Ist vielleicht doch ein bißchen zu schwammig. Da würde ich lieber gar nichts schreiben. Oder wenn schon, dann vielleicht so:

Es ist [mm] ${\sqrt {x^2+1} \over \sqrt {x^3+1} }= \sqrt {x^2+1\over x^3+1}\,.$ [/mm] Der Radikand geht gegen $0$ für [mm] $x\to\infty\,.$ [/mm] Und da die Wurzel stetig ist, strebt sie gegen [mm] $\sqrt [/mm] 0= [mm] 0\,.$ [/mm]

Gruß,
Wolfgang

Bezug
                                                
Bezug
Minimum/Maximum bestimmen: Frage (beantwortet)
Status: (Frage) beantwortet Status 
Datum: 13:11 Sa 19.01.2013
Autor: Blackburn4717537


> Ist vielleicht doch ein bißchen zu schwammig. Da würde
> ich lieber gar nichts schreiben. Oder wenn schon, dann
> vielleicht so:
>  
> Es ist [mm]{\sqrt {x^2+1} \over \sqrt {x^3+1} }= \sqrt {x^2+1\over x^3+1}\,.[/mm]
> Der Radikand geht gegen [mm]0[/mm] für [mm]x\to\infty\,.[/mm] Und da die
> Wurzel stetig ist, strebt sie gegen [mm]\sqrt 0= 0\,.[/mm]
>  
> Gruß,
>  Wolfgang

Okay, aber deine letzte Begründung verstehe ich nicht. Was hat das mit der Stetigkeit der Wurzel zu tun?

Bezug
                                                        
Bezug
Minimum/Maximum bestimmen: Antwort
Status: (Antwort) fertig Status 
Datum: 13:20 Sa 19.01.2013
Autor: Helbig


> > Ist vielleicht doch ein bißchen zu schwammig. Da würde
> > ich lieber gar nichts schreiben. Oder wenn schon, dann
> > vielleicht so:
>  >  
> > Es ist [mm]{\sqrt {x^2+1} \over \sqrt {x^3+1} }= \sqrt {x^2+1\over x^3+1}\,.[/mm]
> > Der Radikand geht gegen [mm]0[/mm] für [mm]x\to\infty\,.[/mm] Und da die
> > Wurzel stetig ist, strebt sie gegen [mm]\sqrt 0= 0\,.[/mm]
>  >  
> > Gruß,
>  >  Wolfgang
>
> Okay, aber deine letzte Begründung verstehe ich nicht. Was
> hat das mit der Stetigkeit der Wurzel zu tun?

Na ja, aus [mm] $x\to [/mm] 0$ folgt [mm] $\sqrt [/mm] x [mm] \to \sqrt [/mm] 0$ doch nur, weil die Funktion [mm] $x\mapsto \sqrt [/mm] x$ in 0 stetig ist. Nimm doch mal eine Funktion [mm] $f\colon[0;\infty)\to\IR$ [/mm] mit $f(0)=1$ und [mm] $f(x)=\sqrt [/mm] x$ für [mm] $x>0\,.$ [/mm] Dann hätten wir $f(0) = 1 [mm] \ne [/mm] 0 [mm] =\lim_{x\to 0} f(x)\;.$ [/mm] Gut, daß wir darüber gesprochen haben!

Gruß,
Wolfgang


Bezug
                                                                
Bezug
Minimum/Maximum bestimmen: Mitteilung
Status: (Mitteilung) Reaktion unnötig Status 
Datum: 02:41 Sa 26.01.2013
Autor: Blackburn4717537

Hallo Wolfgang,

Entschuldigung, dass ich mich jetzt erst melde. War etwas stressig in den letzten Tagen, denn bald fängt die Klausurenphase an. Deine Argumentation habe ich aber verstanden. Danke!

Grüsse
Alexander

Bezug
Ansicht: [ geschachtelt ] | ^ Forum "Stetigkeit"  | ^^ Alle Foren  | ^ Forenbaum  | Materialien


^ Seitenanfang ^
www.englischraum.de
[ Startseite | Forum | Wissen | Kurse | Mitglieder | Team | Impressum ]